ROSH
A 47-year-old man presents to the clinic for evaluation of insomnia, fatigue, abdominal pain, and progressive lower extremity edema over the past nine months. He has a history of hypertension and major depressive disorder. His medications include lisinopril and duloxetine. He does not use tobacco and drinks one to two alcoholic beverages per month. He has a history of intravenous heroin use 18 years ago but does not currently use any illicit drugs. His vital signs are within normal limits. No abnormalities are heard on cardiopulmonary auscultation. His abdomen is soft and nontender with moderate distention. Physical examination is also significant for palmar erythema and pedal edema bilaterally. Abdominal ultrasound shows mild ascites, hypoechoic nodules within the liver, and splenomegaly. Which of the following is the most likely underlying cause for this patient's condition? AAlcoholic liver disease BChronic hepatitis C CNonalcoholic fatty liver disease DNonalcoholic steatohepatitis
B. consistent with hepatic cirrhosis, most likely due to chronic hepatitis C virus (HCV) infection secondary to intravenous heroin use. Physical exam findings may include abdominal distention, enlarged superficial venous vasculature of the abdomen (caput medusa), blood per rectum, spider angiomas, splenomegaly, jaundice, ascites, asterixis, and palpable liver margin. Diagnosis is made with reactive HCV antibody followed by confirmation with HCV RNA testing. Alcoholic liver disease (A) is unlikely based on this patient's limited alcohol consumption and appearance of hypoechoic nodules on ultrasonography. Steatosis is seen in up to 90% of patients with liver disease. Nonalcoholic fatty liver disease (C) and nonalcoholic steatohepatitis (D) appear hyperechoic on ultrasound.
A 50-year-old woman presents with muscle weakness of the shoulder and pelvic girdles that has progressively worsened over the last several months. Physical exam reveals a three out of five muscle strength in the hip flexors and the deltoids. Laboratory studies demonstrate elevated aldolase and creatine kinase. Which of the following positive laboratory results would be expected? AAnti-double stranded DNA antibodies BAnti-Jo-1 antibodies CAnti-La antibodies DAnti-SCL-70 antibodies
BAnti-Jo-1 antibodies Polymyositis-characterized by proximal skeletal muscle weakness and evidence of muscle inflammation difficulty performing the following activities: rising from a chair, climbing stairs, combing hair, lifting arms over the head, and carrying things definitive diagnosis-mm biopsy: muscle fiber necrosis!!, degeneration, regeneration and inflammatory cell infiltrate. first-line treatment is high-dose steroids for at least three months
A 68-year-old man presents to the clinic complaining of worsening chest pain and difficulty breathing. He reports first noticing occasional, brief chest pain and infrequent shortness of breath eight months ago. Physical exam reveals a harsh, crescendo-decrescendo systolic ejection murmur heard over the aorta that radiates into the carotid arteries. He also has a delayed carotid upstroke. Which of the following is the most likely diagnosis based on this patient's history and physical exam findings? AAortic regurgitation BAortic stenosis CMitral stenosis DMitral valve prolapse
BAortic stenosis MR.TRAPS Aortic stenosis is a heart condition caused by thickened, stiff leaflets that result in a narrowing of the aortic valve. This narrowing results in left ventricular outflow obstruction MCC aortic stenosis: rheumatic heart disease & calcification Sx: dyspnea, chest pain, syncope Treatment: aortic valve replacement
What surgical procedure is an alternative to ciprofloxacin to eradicate the carrier state for carriers of Salmonella? ABillroth I BCholecystectomy CSplenectomy DWhipple procedure
BCholecystectomy Cholecystectomy has a higher cure rate and is the preferred treatment in the presence of gallstones but is still not 100% effective at eliminating the carrier state.
A 30-year-old physician assistant student on her pediatrics rotation presents to her primary care provider with a sore throat, tonsillar exudates, anterior cervical lymphadenopathy, abdominal pain, and a subjective fever. Rapid strep testing is positive, and she is treated with amoxicillin. She presents with a fever of 102°F measured in the office 2 weeks later. She has no previous history of rheumatic fever. Which of the following additional findings would be sufficient to make an initial diagnosis of acute rheumatic fever? ACarditis BChorea and reversible prolonged PR interval CErythema marginatum DPolyarthralgia and elevated erythrocyte sedimentation rate
BChorea and reversible prolonged PR interval Acute rheumatic fever (case: fever-minor) Jones criteria - 2 major criteria or 1 major +2minor criteria. major (5): carditis, chorea, subcu nodules, erythema marginatum (nonpruritic rash), polyarthirtis/arthritis Jones criteria: joints, oh no—carditis!, nodules, erythema marginatum, Sydenham chorea minor (4): fever, inc ESR/CRP, prolonged PR interval EKG, polyarthralgia (painful/no inflammation)
A 45-year-old man with progressive dyspnea is given a survival prediction of less than 10 years. His computed tomography scan is significant for diffuse reticular opacities. Considering the advanced stage of his current disease, what other findings would you expect on his physical exam? ABilateral lungs clear to auscultation BClubbing of his fingertips CDecreased FEV1/FVC DPursed lip breathing
BClubbing of his fingertips idiopathic pulmonary fibrosis, which is a chronic progressive interstitial scarring from persistent inflammation. diffuse reticular opacities= "honeycombing," and ground-glass opacities clubbing of the fingers and inspiratory CRACKLES. PFT restrictive. tx;a total lung transplant.
A 65-year-old woman with a history of deep vein thrombosis who is taking warfarin reports to her primary care provider with a three-week history of lower leg swelling, pain, and an itchy rash on the leg that was previously affected by the deep vein thrombosis. Physical exam reveals 2+ pitting edema of the affected leg, brownish discoloration, and a diffuse, flaking, weeping rash over her affected calf. Venous ultrasound does not show new deep vein thrombosis. What initial intervention is recommended? AAltering anticoagulation therapy BCompression stockings CRadioablation therapy DWarm compresses
BCompression stockings chronic venous insufficiency Altering anticoagulation therapy (A) might be indicated if the patient in the above vignette developed a new deep vein thrombosis while on her current medication regimen, but the ultrasound ruled this out. Radioablation therapy (C) is indicated in severe, refractory cases of chronic venous insufficiency. The patient in the vignette has not tried conservative therapy yet, so compression stockings should be utilized initially. Warm compresses (D) are indicated for the treatment of superficial thrombophlebitis, which presents with localized tenderness, pain, and erythema over a palpable vein.
A 65-year-old woman with a history of Hashimoto thyroiditis presents to your office with complaints of an enlarged mass on her neck and unexplained weight loss over the past 3 months. She is now retired but previously worked as a hairdresser and is an avid gardener. Physical exam reveals cervical lymphadenopathy and splenomegaly. Which of the following is the most appropriate diagnostic test? AEndoscopy BExcisional lymph node biopsy CMagnetic resonance imaging DNeck ultrasound
BExcisional lymph node biopsy Fine-needle biopsy is sometimes done initially, but diagnosis requires an accurate histopathologic evaluation that requires sufficient tissue that an excisional biopsy provides Lymphoma is the term used to describe a diverse group of malignancies and is divided into two groups: non-Hodgkin lymphoma and Hodgkin lymphoma. hodgk-reed-sternburg ells. Continuous lymph node spread non-noncontiguous lymph node spread Neck ultrasound (D) does not provide enough information to make the diagnosis of non-Hodgkin lymphoma and is not usually recommended as part of the initial workup.
A 65-year-old man with a history of rheumatic fever presents with decreased exercise tolerance that has progressively worsened over the years. On exam, a low-pitched diastolic murmur is auscultated. The murmur is best heard at the apex with the patient in the left lateral decubitus position. An opening snap is also present. Which of the following is the most common presenting symptom based on the most likely diagnosis? AChest pain BExertional dyspnea CHemoptysis DHoarseness
BExertional dyspnea Mitral stenosis impedes blood flow from the left atrium to the left ventricle, thereby increasing pressure in the left atrium, pulmonary vasculature, and right side of the heart. The majority of cases of mitral stenosis are caused by rheumatic heart disease, resulting in thickening of the leaflets; calcium deposit on valves Mitral Stenosis History of rheumatic heart disease Exertional dyspnea, hemoptysis On auscultation: loud S1 (although intensity diminishes as disease progresses), opening snap, low-pitched rumbling diastolic apical murmur Most commonly caused by rheumatic heart disease Patients with mitral stenosis rarely present with chest pain (A). When chest pain is present, it is most commonly the result of pulmonary hypertension and right ventricular hypertrophy. Mitral stenosis may resemble angina, particularly in the setting of underlying coronary artery disease or coronary artery embolism. Intermittent chest pain may also be caused by atrial tachydysrhythmia with left atrial and pulmonary vascular distention. Hemoptysis (C) may result from mitral stenosis but is less likely than exertional dyspnea. Hemoptysis results from increased pulmonary pressures and vascular congestion. It may present as sudden hemorrhage (due to rupture of thin-walled and dilated bronchial veins), blood-tinged sputum (induced by severe coughing, paroxysmal nocturnal dyspnea, or bronchitis), or frothy pink sputum resulting from pulmonary edema. If the left atrium becomes enlarged, it may compress the recurrent laryngeal nerve, leading to hoarseness (D).
A 20-year-old man presents to his primary care provider in July with complaints of fever, chills, headache, and muscle aches for four days. The patient recalls removing a tick on his arm several days before his symptoms began and is unsure how long the tick had been there. The patient reports that he has been dog sitting for the past week in North Carolina and noticed that one of the dogs had multiple ticks attached. He brought one of the ticks with him to the office today. Which of the following physical exam findings is most consistent with the suspected diagnosis? AErythematous circular rash with central clearing BFacial flushing and abdominal pain CMaculopapular rash on the trunk that spreads to the extremities DTic douloureux
BFacial flushing and abdominal pain American dog tick (Dermacentor variabilis), a vector for the gram-negative coccobacillus Rickettsia rickettsii, which causes Rocky Mountain spotted fever. typically appear 2-14 days after infection and include fever, chills, headache, nausea, vomiting, myalgias, insomnia, and irritability. Facial flushing and abdominal pain are potential signs as well as neurologic signs such as confusion and seizures. The characteristic rash starts as a blanching and maculopapular rash that becomes petechial and begins on the wrists and ankles before spreading to the trunk. RASH typically 2 days AFTER FEVER gold standard diagnostic test is an indirect immunofluorescence assay for the Rickettsia rickettsii antigen tx: DOXY even in child-SE:tooth discoloration and weakened dental enamel
A 58-year-old man is in the cardiac intensive care unit after undergoing PCI for an acute anterior myocardial infarct. The patient reports worsening chest pain 24 hours after arriving in the ICU. When you arrive in the room, the patient is clutching his chest and appears to be in respiratory distress. Vital signs are BP 92/70 mm Hg, HR 102 bpm, RR 28 breaths per minute, and T 98°F. Physical exam reveals elevated JVP. What is the most likely etiology of cardiogenic shock in this patient? ADressler pericarditis BFree wall rupture CMitral valve regurgitation DRecurrent infarction
BFree wall rupture most common types of post-MI mechanical complications are free wall rupture, ventricular septal rupture, and acute mitral valve regurgitation. wall rupture is caused by thinning of the myocardium and stress by asymmetrically contracting cardiac muscles that leads to a tear in the myocardium. symptoms of cardiac tamponade!! (e.g., hypotension, elevated jugular venous pressure, muffled heart sounds [Beck triad], tachycardia, narrowed pulse pressures) seen in an individual within 24-48 hours of MI should raise suspicion for free wall rupture. Dressler pericarditis (A) is a post-MI complication that can lead to cardiac tamponade, however, it classically occurs weeks to months after initial MI. mitral valve regurgitation (C) following MI will typically present with a new cardiac murmur and signs of heart failure. Recurrent infarction (D) should be suspected in a patient with new chest pain and ECG with ST segment elevations following initial reperfusion. Cardiac tamponade is not specific for either mitral valve regurgitation or recurrent infarction and, therefore, is not the most likely cause of this patient's cardiogenic shock.
A 50-year-old man presents to a primary care provider for his annual examination and review of bloodwork. His vital signs are T 37.1°C, BP 150/96 mm Hg, waist circumference 41 inches, body mass index 39 kg/m2, HR 70 bpm, RR 18 breaths per minute, and pulse oximetry 99% on room air. Which of the following additional findings would give this patient a diagnosis of metabolic syndrome? AFasting blood sugar level of 90 mg/dL BHDL level of 35 mg/dL CLDL level of 150 mg/dL DTriglyceride level of 120 mg/dL
BHDL level of 35 mg/dL Metabolic syndrome, also known as syndrome X or insulin resistance syndrome, dx: 3/5 criteria metabolic guidelines 1)waist circumference ≥ 35 inches for women or ≥ 40 inches for men 2) TG level ≥ 150 mg/dL or on medication for hypertriglyceridemia 3) HDL level < 50 mg/dL for women or < 40 mg/dL for men 4) BP ≥ 130/85 mm Hg or on antihypertensive medication 5) fasting blood sugar level ≥ 100 mg/dL or on medication to treat hyperglycemia currently meets two criteria for metabolic syndrome because his waist circumference is 41 inches and his blood pressure is 150/96 mm Hg. A HDL level of 35 mg/dL would give him a diagnosis of metabolic syndrome tx: lifestyle measures (e.g., exercise, diet, weight loss) and pharmacologic treatment of hypertension, dyslipidemia, and hyperglycemia.
A 64-year-old man is diagnosed with pneumonia and admitted into the hospital. Which of the following findings supports Legionella as the pathogen causing pneumonia? AHypoglycemia BHyponatremia CInfiltrate in the upper lobes DPleuritic chest pain
BHyponatremia egionella species are one of the atypical bacterial pathogens that cause pneumonia. Pulmonary exam findings may include decreased breath sounds, dullness to percussion, and rales with auscultation. XRay-confirm alveolar infiltrate However, GI symptoms, such as nausea, vomiting, and diarrhea, are more common in Legionella pneumonia. In patients who have laboratory testing obtained, hyponatremia supports the diagnosis of Legionella pneumonia. empiric regime: levo (fluoroquinolone) or azithromycin* (macrolide) to cover atypical organisms, such as Legionella species
A 72-year-old man with a past medical history significant for hypertension presents for a routine yearly physical exam. He has been feeling well and has no complaints. His physical exam is benign, and a basic metabolic panel is ordered as part of a routine series of blood tests. The results show a calcium level that is 0.8 mg/dL above normal limits. When compared to last year's results, this is a new lab finding. Which of the following studies should be ordered next to confirm the most likely diagnosis? A1,25-dihydroxyvitamin D BIntact parathyroid hormone CPhosphate DThyroid ultrasoun
BIntact parathyroid hormone ↑ PTH, ↑ Ca, ↓ phosphorus Primary hyperparathyroidism describes a condition where the regulation of serum ionized calcium by parathyroid hormone is abnormal. Normally, parathyroid hormone is secreted in response to lower calcium levels. This hormone acts to increase bone resorption, increase intestinal calcium absorption, and decrease urinary calcium excretion to raise serum calcium levels back into the normal range and maintain homeostasis. In primary hyperparathyroidism, there is hypersecretion of parathyroid hormone and, therefore, increased serum calcium levels. Measuring intact parathyroid hormone confirms the diagnosis. Other testing includes a 24-hour urine calcium test to assess renal function and genetic testing definitive management of primary hyperparathyroidism is through a parathyroidectomy (ONLY)
A 12-year-old boy presents to your office for a routine sports physical exam. Family history reveals the patient had an uncle who died of a sudden unexplained cardiac event. You obtain a screening ECG, which is shown above. Which of the following best describes the pathophysiology of the suspected diagnosis? AAccessory conduction pathway BAtrioventricular nodal reentrant pathway CDecreased atrioventricular node transmission DMultiple atrial electrical foci
AAccessory conduction pathway ECG will show short PR interval, delta wave, wide QRS Wolff-Parkinson-White syndrome is caused by an accessory conduction pathway formed between the ventricles and atria of the heart. In Wolff-Parkinson-White syndrome, an accessory conduction pathway (also known as the bundle of Kent) allows the electrical impulse to bypass the AV node and, thus, cause preexcitation of the ventricular myocardium. stabilized with an antidysrhythmic agent=procainamide, the definitive treatment is ablating the accessory pathway. AV nodal reentrant pathways (B) are the cause of paroxysmal supraventricular tachycardia. Paroxysmal supraventricular tachycardia occurs when an additional electrical conduction pathway is formed within the AV node. This new pathway causes a reentrant tachycardia. Decreased AV node transmission (C) is the cause of AV node block. AV nodal block occurs when the electrical impulse originating from the SA node is slowed or not transmitted through the AV node. AV nodal block results in mild to severe bradycardia, depending on the extent of the block. Multiple atrial electrical foci (D) cause atrial fibrillation. Atrial fibrillation causes tachycardia due to excitable atrial foci that emit electrical impulses at varying rates.
A 57-year-old female athlete presents to a primary care provider with a fever, maculopapular rash, and arthralgias. She reports she has been working out for 3 hours per day but has not increased the duration or frequency of her workouts in the last year. The only change she can recall is the use of ibuprofen 800 mg three times daily over the last 2 weeks for a mild quadriceps strain. Urinalysis is significant for proteinuria, eosinophiluria, increased white blood cells, and white cell casts. No red blood cells are seen on urine microscopy. What is the most likely diagnosis? AAcute interstitial nephritis BPyelonephritis CRhabdomyolysis DSystemic lupus erythematosus
AAcute interstitial nephritis Acute interstitial nephritis is characterized by an abrupt decline in kidney function due to damage or inflammation of the renal tubules and interstitium antibiotics, nonsteroidal anti-inflammatory drugs (NSAIDs), sulfonamides, diuretics, and antituberculous drugs. NSAIDs (e.g., ibuprofen) are the most frequent cause of drug-induced acute interstitial nephritis urinalysis will contain white blood cells, red blood cells, white cell casts, and occasionally eosinophils.' Kidney biopsy is used for definitive diagnosis if the diagnosis is clinically unclear Pyelonephritis (B) is an infectious inflammatory condition affecting the kidney parenchyma and renal pelvis. The classic triad is fever, costovertebral angle tenderness, and nausea or vomiting. A maculopapular rash is not usually seen with pyelonephritis
A 65-year-old woman who is a former smoker with a past medical history of hypertension, hyperlipidemia, and arthritis presents to her primary care clinic for three months of cough with hemoptysis, generalized chest discomfort, and weight loss. Vitals show a heart rate of 68 bpm, blood pressure of 132/81 mm Hg, O2 saturation of 96% on room air, respiratory rate of 16 breaths per minute, and temperature of 98.1°F. Vaccines are up to date, and there has been no travel or sick contacts. Chest X-ray and CT scan of the chest show multiple irregularly shaped nodules in the periphery of the lungs bilaterally. Which of the following is the most likely diagnosis? AAdenocarcinoma BLarge cell carcinoma CSmall cell lung cancer DSquamous cell carcinoma
AAdenocarcinoma Large cell lung cancer (B) is a less common carcinoma that can present centrally or peripherally with a rapid doubling time and early metastasis. Small cell lung cancer (C) originates in the central bronchi and will present as a central mass. Squamous cell carcinoma (D) is bronchial in origin and presents with a central mass primarily.
A 78-year-old woman with metastatic breast cancer presents with nausea, anorexia, and generalized weakness for the past several days. A basic metabolic panel reveals a serum creatinine of 0.9 mg/dL and a serum calcium of 14.2 mg/dL. Which of the following interventions should be done immediately? AAdminister intravenous saline BBegin administering oral bisphosphonate CPrepare patient for hemodialysis DPrescribe oral glucocorticoids
AAdminister intravenous saline Hypercalcemia=Bones, stones, groans, psychiatric overtones. Sym Malignancy and primary hyperparathyroidism are the two most common causes bone pain (bones), kidney stones (stones), abdominal pain (groans), lethargy, psychosis (psychiatric overtones) ECG: shortened QT interval Hypovolemic states will prevent renal excretion of calcium, so volume expansion using intravenous saline is recommended. These patients should be treated with IV saline, calcitonin, and an IV bisphosphonate. Patients who have hypercalcemia secondary to malignancy typically require maintenance therapy with a bisphosphonate derivative. Patients with chronic granulomatous disease may have an overproduction of calcitriol. These patients typically benefit from glucocorticoids to decrease calcitriol production. severe renal disease tx Denosumab instead bisphosphonate dialysis if hypercalcemis >18
A 32-year-old woman is being evaluated for infertility. She also complains of headaches and vision disturbance. On exam, she is noted to have bitemporal hemianopsia. Laboratory studies reveal a serum prolactin level of 210 ng/mL. Which of the following history findings would most likely be present? AAmenorrhea BDysmenorrhea CIncreased libido DRegular menses
AAmenorrhea pituitary adenoma is a slow-growing abnormal growth in the pituitary gland Pituitary adenoma can be functional (hormone-secreting) or nonfunctional (non-hormone-secreting) and microadenoma (< 1 cm in size) or macroadenoma (> 1 cm in size). pituitary adenoma that secretes prolactin is called prolactinoma-more commonly affects premenopausal women (women of reproductive age) Prolactin is a hormone that stimulates milk production. Prolactinoma can cause a mass effect or hyperprolactinemia or both. Clinical features of prolactinoma due to hyperprolactinemia include galactorrhea, amenorrhea, irregular menses, and infertility, loss of libido. When prolactinoma causes a mass effect, headache and vision loss (bitemporal hemianopsia) occur. amenorrhea, hot flashes, and vaginal dryness. MRI may show a sellar lesion first-line treatment is dopamine agonists (bromocriptine, cabergoline). Surgical resection is indicated in adenomas that are refractory to first-line therapy
A 52-year-old man with a history of diabetes mellitus type 2 and cigarette smoking presents to your clinic with a six-month history of worsening leg pain. The patient reports his pain is worse when walking and improved with rest. Physical exam reveals decreased hair growth over the distal extremities and bilateral diminished dorsalis pedis pulses. What is the most appropriate diagnostic test to establish the suspected diagnosis? AAnkle-brachial index BCTA of the lower extremities CDuplex ultrasonography DMRA of the lower extremities
AAnkle-brachial index Peripheral arterial disease (PAD) is the atherosclerotic or thromboembolic obstruction of the peripheral arteries. present with exertional pain that improves with rest=claudication nonhealing ulcers (usually located on the lateral malleolus), muscle atrophy, diminished hair growth in the distal extremities, vascular bruits, thick toenails, and decreased peripheral pulses. ABI) testing is the initial test=ratio of systolic blood pressure in the dorsalis pedis artery to systolic blood pressure in the brachial artery. ABI of 0.9-1.3 is normal. ABI of less than 0.9 is indicative of mild to moderate All patients with PAD should be treated with a statin agent, an antiplatelet agent, and lifestyle modification encouragement (e.g., smoking cessation, exercise, and appropriate diet) Primary risk factors for developing PAD include hyperlipidemia, hypertension, diabetes, and tobacco use.
A 76-year-old woman with congestive heart failure presents with palpitations and dyspnea. On exam, her pulse is irregular, with a rate of 78 beats/minute. Crackles at the lung bases are also present on lung auscultation. Her rhythm strip is shown above. Which of the following medications should be initiated? AApixaban BAspirinYour Answer CEnalapril DVerapamil
AApixaban = Atrial fibrillation is the most common sustained dysrhythmia in adults. Risk factors include hypertension, obesity, sleep apnea, hyperthyroidism, diabetes, ischemic heart disease, valvular disorders, excessive alcohol consumption, heart failure, and cardiomyopathy. Rate control can be achieved by administering atrioventricular nodal blockers such as beta-adrenergic blockers (most effective) and nondihydropyridine calcium channel blockers (such as verapamil and digoxin). Unstable: cardioversion Stable: rate control is mainstay (diltiazem, metoprolol) Rhythm control is indicated in patients with continuing symptoms despite rate control, whose rate is difficult to control, or who develop tachycardia-mediated cardiomyopathy or in younger patients with the first onset of the disease or whose symptoms are precipitated by an acute illness. Cardioversion, antidysrhythmic medications, and catheter ablation can be used for rhythm control. Patients with atrial fibrillation have a fivefold risk of developing stroke. Thus, the need for anticoagulation should be determined using the CHA2DS2-Vasc score. Patients with a score of 2 or above should be started on anticoagulation such as rivaroxaban, dabigatran, or apixaban. Aspirin (B) is an antiplatelet drug used to prevent stroke and myocardial infarction. It has no role in anticoagulation therapy used for stroke prevention in patients with atrial fibrillation. Enalapril (C) is an angiotensin-converting enzyme inhibitor that can treat hypertension but has no role in stroke prevention. Verapamil (D) is a nondihydropyridine drug that can be used for rate control in patients with atrial fibrillation whose resting heart rate is greater than 80 beats/minute or whose exertional heart rate is greater than 100 beats/minute. It should not be used, however, in patients with systolic heart failure.
A 42-year-old man presents to a primary care provider with sudden-onset high fever, chills, dysuria, and perineal pain. Digital rectal examination reveals the presence of a warm, exquisitely tender prostate. The patient is prescribed ciprofloxacin. His fever is resolved the next day. Which of the following best describes the most common route of infection of the patient's condition? AAscending urethral spread BDirect extension from the bladder CHematogenous spread DLymphatic spread from the rectum
AAscending urethral spread acute bacterial prostatitis-route of infection is ascending urethral spread or reflux of urine into the prostatic ducts. Urinalysis will show pyuria, bacteriuria, or hematuria. A urine culture is the definitive < 35 years old: N. gonorrhoeae, C. trachomatis-ceftriaxone IM and doxycycline > 35 years old: E. coli-cipro/levo=fluoroquinolone or TMP-SMX for 4 weeks pelvic CT or transrectal ultrasound for a prostatic abscess is indicated in patients who do not respond to antibiotics within 48 hours.
A 42-year-old woman presents with vague complaints of fatigue and unexplained weight gain for the last several months. A thyroid-stimulating hormone assay is elevated, and free T4 is low. Which of the following physical exam findings would be most consistent with the diagnosis? ABrittle fingernails BOphthalmopathy CPitting edema DSkin hyperpigmentation
ABrittle fingernails Sweating can be decreased, fingernails become brittle, and hair may become coarse and dry. Patients may complain of general thinning of the hair. Examination of the eyes may reveal periorbital edema. Decreased heart rate tx: levothyroxine Skin hyperpigmentation (D) is associated with severe hyperthyroidism and is secondary to accelerated cortisol metabolism leading to an increase in corticotropin secretion.
A 65-year-old man with a past medical history of chronic kidney disease stage IV presents to the clinic to discuss his lab results. The patient does not report any symptoms or concerns at this time. His BMP results include glucose 140 mg/dL, creatinine 3.86 mg/dL, eGFR 21 mL/min, sodium 142 mmol/L, potassium 7.2 mmol/L, chloride 108 mmol/L, carbon dioxide 21 mmol/L, and calcium 9.7 mmol/L. The patient's ECG is noted above. Which of the following should be administered as the most appropriate next step in this patient's management? ACalcium gluconate BInsulin CKayexalate DSodium bicarbonate
ACalcium gluconate #1=stabilize pt's cardiac function for tx. Diagnosis of hyperkalemia is based on a serum potassium level greater than 5 calcium 4 to 5.6 ekg-elevated or peaked T waves, widening of the QRS complex, and PR prolongation. immediately administer calcium gluconate or calcium chloride to stabilize the myocardium and increase the cardiac threshold. The second step is intended to decrease serum potassium by redistributing the potassium into the cells, which can be done by administering insulin, albuterol, sodium bicarbonate, or a combination of these. The final step in management is to remove the potassium from the body by administering loop diuretics or sodium polystyrene. Insulin (B) and sodium bicarbonate (D) can be used as the second step in management for patients with hyperkalemia. History of kidney failure, DKA, rhabdomyolysis, tumor lysis Lethargy, weakness, paralysis PE will show bradycardia, hypotension, cardiac dysrhythmia
A 33-year-old woman with a long history of recurrent abdominal pain and diarrhea presents to the office with increasing pain and pressure to the rectal area for about three weeks. She was diagnosed with an abscess to the buttocks about two weeks ago and given a course of antibiotics. However, she continues to have throbbing pain in the area that is worse with sitting and defecating. She complains of a new symptom of foul-smelling perianal leakage for about three days. Upon physical exam, there is induration to the perianal area with a small opening that exudes purulent fecal material when expressed. Which of the following diseases are her symptoms classically associated with? ACrohn disease BHidradenitis suppurativa CIrritable bowel syndrome DUlcerative colitis
ACrohn disease =-Any part from the mouth to the anus. Irritable bowel syndrome (C) is characterized by a combination of constipation, abdominal pain, bloating, and diarrhea but is not associated with the development of anal abscesses and fistulas. Unlike Crohn disease, which affects the full thickness of the gastrointestinal wall, ulcerative colitis (D) is not associated with anal fistulas since it only affects the gastrointestinal mucosa.
A 23-year-old man presents to your clinic with a two-day history of worsening parasternal chest pain. The patient reports the pain is sharp and made worse with breathing and lying down. Additional symptoms include a nonproductive cough, which the patient states started several days before the onset of his chest pain. Vital signs are within normal limits. Cardiopulmonary exam is unremarkable. You obtain an ECG and decide to treat this patient with high-dose aspirin. Which of the following ECG findings is most indicative of the suspected diagnosis? ADiffuse ST segment elevations BNormal sinus rhythm CSinus tachycardia DT wave inversions
ADiffuse ST segment elevations Acute pericarditis occurs when the pericardial sac becomes inflamed. T wave inversions (D) are a common finding in patients with acute pericarditis, however, T wave inversions usually develop one to two weeks after disease onset. PE will show tachycardia and pericardial friction rub ECG will show PR depression, PR elevation (aVR), diffuse ST segment elevation (concave) Most commonly caused by idiopathic then viral (coxsackie) Treatment is NSAIDs, colchicine
A 72-year-old woman presents to the office complaining of worsening aching, throbbing, and heaviness in both legs. She also reports frequent night cramps in her legs and feeling like she cannot get comfortable while falling asleep. On physical exam, both lower extremities appear edematous with scattered patches of reddish-brown hyperpigmentation. On her distal lower legs bilaterally, there is a fibrosing panniculitis with hyperpigmentation and induration involving most of the leg circumferentially. Her left medial ankle is edematous and has a 7 mm shallow ulcer with a red base and irregular borders. Doppler ultrasound reveals a reflux of 1005 milliseconds in the left great saphenous vein and 1007 milliseconds in the right. Considering this information, what would be the most effective clinical intervention for this patient? AEndovenous laser ablation BSclerotherapy CSurgical vein stripping DVenous reconstruction
AEndovenous laser ablation Chronic venous insufficiency is a disease of variable severity caused by venous hypertension and incompetent valves leading to dilated veins, discoloration, swelling, pain, skin changes, and ulceration. duplex ultrasound scanning is the diagnostic modality of choice. high ligation and stripping of the saphenous vein was the preferred surgical treatment. Currently, this technique has been replaced by percutaneous endovenous thermal ablation. This procedure is performed on an ambulatory basis with local anesthesia. Patients are typically completely ambulatory following treatment, and the recovery time is short. Two types of thermal ablation procedures exist: endovenous laser ablation and radiofrequency ablation. Significant reflux, defined as reflux greater than 1000 ms in superficial veins (such as the great saphenous vein described in the above vignette), documented on Doppler ultrasound examination is an indication for endovenous laser ablation. For telangiectasias and reticular veins that are asymptomatic but cosmetically bothersome, sclerotherapy (B) or laser therapy may be effective. The results of sclerotherapy are generally thought to be superior to those of laser therapy. Patients with isolated varicose veins without truncal reflux are typically treated with sclerotherapy or excision depending on the size, location, and number of veins involved. Surgical vein stripping (C), also known as high ligation and stripping, has been largely replaced by endovenous laser ablation due to the ease and relatively low-risk profile of the latter procedure. Venous reconstruction (D) is typically reserved for patients with severe vascular damage, particularly after a deep vein thrombosis. It is not necessary in this case.
A 68-year-old man presents to the clinic complaining of abdominal distention and shortness of breath. He takes no daily medications, has not seen a healthcare provider in decades, and has consumed 12 beers per day for the past 20 years. On physical exam, vital signs are within normal limits. Skin is nonicteric, and body habitus demonstrates central adiposity. Physical exam reveals a firm, irregular liver border just below the right costal margin, abdominal fluid wave, and dullness to percussion with decreased breath sounds on the right chest. Which of the following represents appropriate initial management of this patient's complaints? AFurosemide 20 mg daily and salt restriction BInsertion of a thoracostomy tube in the right chest and abdominal paracentesis CLiver transplantation DMetoprolol tartrate 50 mg bid and potassium restriction
AFurosemide 20 mg daily and salt restriction Patients with ascites can be treated with diuretics such as furosemide and salt restriction Cirrhosis of the liver describes hepatic fibrosis and regenerative nodule formation in the presence of chronic hepatic insults. The most common of these insults are alcohol, fatty liver disease, and chronic viral hepatitis. ascites, platelets under 160,000/µL, spider angiomata, a diminished ALT:AST ratio, and a prolonged international normalized ratio. Hepatomegaly may be evident on exam, although advanced cirrhosis can lead to liver consolidation. The liver border may feel nodular on exam. Complications of cirrhosis are many and include ascites, spontaneous peritonitis, hepatorenal syndrome, hepatic heart failure, pleural effusion (generally on the right), hepatic encephalopathy, splenomegaly, bleeding, variceal formation, and variceal rupture. Patients with cirrhosis must be counseled to abstain from alcohol, hepatotoxic medications such as acetaminophen, and certain hepatotoxic supplements and herbal remedies. cirrhosis & ascities=1) hx 2) PE 3) abd US/abd paracentesis -> labs
A 37-year-old woman presents to a primary care provider with anxiety, difficulty sleeping, and vision changes for the past three months. Her family history is significant for a mother and grandmother with Hashimoto thyroiditis. She was diagnosed with atrial fibrillation two years ago and has been taking amiodarone. Her vital signs are T 37.1°C, BP 122/72 mm Hg, HR 56 bpm, RR 18 breaths per minute, and pulse oximetry 99% on room air. Physical examination reveals superior lid lag with a downward gaze and eyelid retraction. The patient is unable to look upwards when testing extraocular movements. A nontender, diffusely enlarged thyroid is palpated on exam. Laboratory testing reveals elevated levels of antithyrotropin receptor antibodies, thyroid-stimulating immunoglobulin, antithyroglobulin antibodies, and antithyroid peroxidase antibodies. A radioactive iodine uptake study is ordered and shows a diffuse** increased uptake in the thyroid tissue. What is the most likely diagnosis? AGraves disease BHashimoto thyroiditis CSubacute thyroiditis DToxic multinodular goiter
AGraves disease +sym: Pretibial myxedema (bilateral, non-pitting, yellow or brown papules, nodules, and plaques over the shins) Amiodarone use increases the risk for developing Graves disease because it contains iodine. Thyroid receptor antibodies are the most specific test tx: symp-beta-blocker (e.g., propranolol). Long-term pharmacologic management of Graves disease includes thiourea medications such as propylthiouracil (PTU) or methimazole. Toxic multinodular goiter (D) is the second most common cause of hyperthyroidism after Graves disease. Antiperoxidase antibodies and antithyroglobulin antibodies will be absent in toxic multinodular goiter. Radioactive iodine uptake testing will show pockets of increased iodine uptake rather than the diffuse increased uptake seen in Graves disease.
A 25-year-old man presents to the clinic with concerns about pain in both eyes, right knee pain, and burning in his genitalia. He reports he is sexually active and recently was treated with antibiotics for a sexually transmitted infection. On physical exam, you note bilateral conjunctivitis, erythema of the right knee, and tenderness over the joint line of the knee. Genital examination reveals urethral irritation present at the meatus. Vitals are within normal limits. Which allele is implicated in the most likely diagnosis? AHuman leukocyte antigen B27 BHuman leukocyte antigen B47 CHuman leukocyte antigen DR2 DHuman leukocyte antigen DR3
AHuman leukocyte antigen B27 Reactive arthritis is a spondyloarthropathy that results most commonly after a sexually transmitted infection (STI) or gastrointestinal illness. It is associated with HLA-B27. Chlamydia trachomatis is the most commonly associated sexually transmitted infection. Risk factors: male sex, HLA-B27+, history of recent GI or Chlamydia infection Sx: acute asymmetric arthritis PE: conjunctivitis, arthritis, urethritis, keratoderma blennorrhagicum (lesions on palms and soles) Labs: may show positive stool cultures or + Chlamydia NAAT Tx: NSAIDs Mnemonic: can't see (conjunctivitis, uveitis), can't pee (urethritis), can't climb a tree (arthritis) if reactive arthritis is unresponsive to methotrexate and sulfasalazine? Tumor necrosis factor inhibitors such as adalimumab or etanercept.
A 38-year-old woman has been diagnosed with papillary thyroid cancer. The tumor is 2.3 cm in diameter with metastasis to regional lymph nodes and no distant metastasis. Which of the following best describes the stage of this cancer? AStage I BStage II CStage III DStage IV
AStage I Papillary thyroid cancer is a thyroid follicular epithelial-derived cancer that is considered a differentiated type of cancer complaints of a thyroid nodule or enlarged ipsilateral lymph node, tumor, node, metastasis (TNM) system for staging is typically used for papillary thyroid cancer Stage I <45 yo, then patients with any tumor size and any node metastasis without distant metastasis!!! Stage II <45yo If there is distant metastasis Stage III (C) >45yo with a tumor that extends into the adjacent soft tissues without distant metastasis. Stage IV (D) > 45yo when the tumor invades prevertebral fascia or encaseates the carotid without metastasis or when there is any tumor or node involvement with distant metastasis.
first line recurrent absence seizures?
Answer: Ethosuximide.
A 40-year-old woman presents to her primary care provider with complaints of hand pain. She states her wrists and hands hurt and feel very stiff every morning. She does not notice the pain as much later in the day after she has been moving around for a while. Physical exam reveals ulnar deviation at the metacarpophalangeal joint bilaterally, as well as swollen, tender interphalangeal joints. What physical exam finding is most likely, given the suspected diagnosis? AHard, fixed nodules at the distal interphalangeal joint BHyperextension of the distal interphalangeal joint with flexion of the proximal interphalangeal joint CInability to extend the distal interphalangeal joint DUnstable first metacarpophalangeal joint in the neutral position
B Hyperextension of the distal interphalangeal joint with flexion of the proximal interphalangeal joint = boutonnière deformity swan neck deformity-flexion at the distal interphalangeal joint and hyperextension at the proximal interphalangeal joint ulnar deviation at the metacarpophalangeal joint is a classic physical exam Rheumatoid arthritis (RA)-Morning stiffness lasting > 30 minutes tx-initiation of disease-modifying antirheumatic drugs (DMARDs) Nonbiologics include methotrexate, hydroxychloroquine, and sulfasalazine Biologic options include etanercept, adalimumab, rituximab, and infliximab. All patients started on biologic DMARDs need to be tested for TB, hepatitis B, and hepatitis C prior to initiation NSAID-first line PAIN control
A 63-year-old man with a history of Barrett esophagus presents to the office with concerns of progressively worsening dysphagia over the past few months. He reports the dysphagia initially seemed to affect only solid foods, but now it is starting to impact thicker liquids. Sometimes he feels that food gets stuck in his throat and upper chest when swallowing. When comparing his anthropometrics from his previous visit 6 months ago, it is apparent he has lost 17 pounds. On physical examination, he is also noted to have a hoarse voice, which he reports has been present for the past 2 months. Which of the following diagnoses is of greatest concern given his presentation? ACalcinosis, Raynaud phenomenon, esophageal dysmotility, sclerodactyly, and telangiectasia syndrome BEsophageal cancer CMyasthenia gravis DZenker diverticulum
B. Esophageal cancer is suspected in patients with Barrett esophagus who have progressive dysphagia and weight loss
A 45-year-old man presents to his primary care provider with a sudden onset of left-sided facial weakness and severe left ear pain. He reports that he woke up this morning and was unable to close his left eye. In addition to his ear pain, he reports increased sensitivity to sound on the left side. On physical examination, the patient is unable to close his left eye, raise his left eyebrow, or smile on the left side. He has a history of diabetes mellitus and recurrent fever blisters. His left external ear canal is normal. Which of the following interventions is most likely to increase the patient's chance of a full recovery? ALubricant eye drops BOral prednisone CPhysical therapy DValacyclovir
BOral prednisone Bell palsy caused by herpes simplex virus important differentiating factor between Bell palsy and a stroke is that stroke patients will be able to raise their eyebrows and wrinkle their forehead. protecting the affected eye with lubricating eye drops or patching and corticosteroids (e.g., prednisone 25 mg PO bid for 10 days). Corticosteroids increase the chance of complete recovery at 9-12 months Waking up with unilateral facial nerve paralysis, hyperacusis, and taste disturbance PE will show CN VII palsy that does NOT spare the forehead
A 70-year-old woman with a known history of congestive heart failure presents to the clinic with pleuritic chest pain. On physical exam, the patient has decreased tactile fremitus, decreased breath sounds, and dullness to percussion to bilateral lower lung bases. A chest X-ray is significant for blunting of costophrenic angles. A thoracentesis is performed and the pleural fluid is sent for further evaluation and chemistry. Which of the following findings would you expect to see in the patient's fluid analysis? APleural fluid lactate dehydrogenase > two-thirds the upper limit of normal lactate dehydrogenase BPleural fluid lactate dehydrogenase: serum lactate dehydrogenase = 0.5 CPleural fluid lactate dehydrogenase: serum lactate dehydrogenase = 0.7 DPleural fluid protein: serum protein = 0.6
BPleural fluid lactate dehydrogenase: serum lactate dehydrogenase = 0.5 patient with congestive heart failure is likely to have transudative pleural effusion. A,C,D=exudative pleural effusion
A 68-year-old woman presents to urgent care with a rapid onset of symmetrical pain and stiffness of the shoulders, neck, and hip girdle that is worse in the morning and after prolonged inactivity. Because of the stiffness and pain, she is having difficulty with daily activities, such as combing her hair, taking a shower, putting on a coat, and driving. Which of the following is the most likely explanation for these findings? AGiant cell arteritis BPolymyalgia rheumatica CPolymyositis DTakayasu arteritis
BPolymyalgia rheumatica Proximal muscle (shoulders, neck, hip girdle) stiffness and aching, especially in the morning, that improves during the day but worsens after inactivity symmetrical decreased range of motion in the neck, hips, and especially in the shoulder girdle Anemia and elevated ESR >40 tx-oral low dose corticosteriod and taper associated with giant cell (temporal) arteritis Polymyositis (C) is an inflammatory disease presenting with subacute proximal muscle weakness affecting both sides of the body. Most commonly, women are affected more than men, and symptoms typically develop gradually over weeks and months
A 24-year-old man with Crohn disease presents to the clinic with abdominal pain and is found to have three strictures within a short segment of the proximal small bowel on endoscopic evaluation. The patient has not undergone any previous surgical interventions. Which of the following is the appropriate clinical intervention for this patient? ABalloon dilatation BSmall bowel resection with anastomosis CSmall bowel resection with stoma DStrictureplasty
BSmall bowel resection with anastomosis Any part of the gastrointestinal tract, from mouth to anus, can be involved. Colonoscopy will show skip lesions, cobblestone mucosa, transmural inflammation. definitive treatment for any of the above complications is bowel resection. Surgery will NOT be curative Patients with multiple strictures in a short segment of bowel, such as the patient in the above vignette, benefit most from bowel resection with surgical anastomosis. Small bowel resection with stoma (C) is not appropriate because the lesion is in the proximal small bowel and would necessitate removal of the majority of the intestines. ASCA positive, pANCA negative
A 65-year-old man with a past medical history of non-small cell lung cancer who was treated with radiation therapy and chemotherapy presents to the emergency department with a three-day history of shortness of breath, fatigue, and swelling in his legs. Physical exam reveals hypotension, elevated jugular venous distension, and 2+ pitting edema bilaterally. Echocardiogram reveals a thickened pericardium. Which of the following physical exam findings would you expect to find, given the suspected diagnosis? ADisplaced point of maximal impulse BMuffled heart sounds CPericardial knock DS3 gallop
C constrictive pericarditis. Constrictive pericarditis is a thickened, fibrotic pericardium that restricts ventricular diastolic filling. atients with constrictive pericarditis present with dyspnea, fatigue, and signs of right-sided heart failure, such as peripheral edema. PE =elevated regular venous distention, hepatojugular reflux, and a pericardial knock. pericardial knock is heard when ventricular filling suddenly ceases due to the thickened and noncompliant pericardium. If the echocardiogram is nondiagnostic, a right-sided heart catheterization may be done hemodynamically stable patients is similar to pericarditis treatment and includes colchicine and a nonsteroidal anti-inflammatory agent. Diuretics are also commonly utilized in the management of constrictive pericarditis. Treatment of hemodynamically unstable patients or patients with persistent symptoms involves a pericardiectomy.
A 55-year-old man presents to his primary care provider for palpitations and mild dyspnea. His past medical history is significant for dilated cardiomyopathy. An ECG done in the office is shown above. Vital signs in the office are T 98.8°F (37.1°C), BP 116/70 mm Hg, HR 165 bpm, and RR 22 breaths per minute. He is taken to the emergency department for a full cardiac workup. His ECG is unchanged from earlier. CBC, CMP, magnesium, and serial troponins are within normal limits. The patient is alert and oriented. Vitals are T 98.8°F (37.1°C), BP 122/72 mm Hg, HR 170 bpm, RR 24 breaths per minute, and pulse oximetry 97% on room air. Which of the following is the most appropriate next step in management? AAmiodarone 300 mg IV over two minutes BCatheter ablation CProcainamide 100 mg IV over two minutes DSynchronized cardioversion Vtach tx for stable pt?
C. procainamide 100 mg IV over two minutes in the treatment of stable monomorphic VT Pulseless: immediate defibrillation Unstable: synchronized cardioversion Stable: procainamide, amiodarone, synchronized cardioversion (refractory) SVT is amiodarone 150 mg IV over 10 minutes. If VT recurs after synchronized cardioversion, IV amiodarone should be given with additional cardioversion attempts as necessary.
Sjögren syndrome is an autoimmune condition caused by inflammation and destruction of exocrine glands. complication can lead to?? ADeep vein thrombosis BGastric carcinoma CNon-Hodgkin lymphoma DPulmonary hypertension
C.Non-Hodgkin lymphoma Sjögren syndrome Autoimmune disorder most common in women 40-60 years old Sx: dry eyes (xerophthalmia) and dry mouth (xerostomia) Labs: SSA (anti-Ro) or SSB (anti-La) Diagnosis: Schirmer test, salivary gland biopsy Higher risk of lymphoma, interstitial lung disease, type 1 RTA
A 50-year-old man presents to the clinic for results of his first screening colonoscopy. He takes no medications, has no symptoms, and has a negative family history of colorectal cancer. On colonoscopy, two villous adenomatous polyps of 1 cm each were removed and no other lesions noted. Which of the following is the correct interval between future surveillance colonoscopies for this patient, according to the United States Preventive Task Force? A1 year B10 years C3 years D5 years
C3 years Patients with high-risk polyps should receive surveillance colonoscopy in 3 years. This includes patients with three or more adenomas, with tubular adenomas that are 10 mm or larger, with villous adenomas, or with high-grade dysplasia Surveillance every 10 years (B) is recommended for anyone who has had a negative initial screening colonoscopy. Surveillance every 5 years (D) can be appropriate for patients with one or two tubular adenomas under 10 mm discovered on initial screening colonoscopy. Surveillance every 1 year (A) for colon cancer is appropriate for patients with over 10 to 20 adenomas on initial colonoscopy or familial adenomatous polyposis or Lynch syndrome. One first-degree relative with colon cancer or advanced polyp: colonoscopy at age 40 or 10 years earlier than the age at which relative diagnosed Familial adenomatous polyposis: sigmoidoscopy at age 12, then every 1-2 years
A 61-year-old woman presents to the emergency department with fever and abdominal pain for the past 2 days. She reports mild nausea but no episodes of emesis. She notes a previous history of an "abnormal colonoscopy". On physical examination, she is febrile to 100.8°F and is tender in the left lower quadrant. A mass is palpable in the left lower quadrant. Which of the following diagnostic studies would have the highest sensitivity and specificity for confirming the diagnosis, given her most likely diagnosis and associated complication? AAbdominal radiograph BAbdominal ultrasound CCT scan of the abdomen DMRI of the abdomen
CCT scan of the abdomen Diverticulitis abscess may result in a palpable mass in the left lower quadrant. CT scan is the preferred diagnostic study for diagnosis and demonstrates bowel wall thickening, colonic diverticula, Diagnosis is made by CT with IV contrast: thickened bowel wall, "fat stranding," may show complications - bowel perforation, abscess, fistula, obstruction
A 32-year-old woman presents to the clinic for a three-month follow-up. She is currently taking loperamide 2 mg and hyoscyamine 0.25 mg 45 minutes before each meal for irritable bowel syndrome with predominant diarrhea. She states the diarrhea is well controlled, but her abdominal pain and bloating continue unabated. Which of the following is the best therapeutic choice for this patient? AAdd alprazolam 0.25 mg tid to current regimen BAdd dicyclomine 20 mg qid to current regimen CDiscontinue hyoscyamine and add amitriptyline 12.5 mg qhs DDiscontinue loperamide and add rifaximin 550 mg tid
CDiscontinue hyoscyamine and add amitriptyline 12.5 mg qhs Loperamide working well for diarrhea so don't d/c. Rifaximin is an antibiotic that is currently used to treat abdominal pain and bloating in patients who have failed antispasmodics, antidepressants, and dietary modification. Irritable bowel syndrome is defined as abdominal pain and bloating at least one day of every week for three months or more associated with pain on defecation, change in frequency of stools, or change in consistency of stools that cannot be attributed to any other colonic disorder. diarrhea-predominant symptoms should be started on loperamide 2 mg 45 minutes before each meal. Those with mixed diarrhea and constipation must use loperamide on an as-needed basis and judiciously. Patients with constipation-predominant symptoms can be prescribed polyethylene glycol as a first-line agent. The second aim of treatment in irritable bowel syndrome is relief of abdominal pain and bloating. First-line therapy is with diet modification-pharmacologic therapy should begin with antispasmodic agents. These include hyoscyamine and dicyclomine. If a patient fails to find relief of symptoms with an antispasmodic, the second-line therapy is a tricyclic antidepressant, such as amitriptyline, nortriptyline, desipramine, or imipramine.
A 40-year-old woman presents to the clinic reporting three weeks of worsening chest discomfort and dyspnea. The patient notes her dyspnea worsens when lying down at night or when she lies on her recliner to watch television. An echocardiogram is done at the bedside and is significant for fluid within the pericardial sac. Cardiac function remains intact, and there is no change in ventricular filling. Which of the following is most likely going to be found on this patient's physical exam? AA blood pressure reading of 88/48 mm Hg BA pericardial knock CDistant heart sounds DJugular venous distension
CDistant heart sounds pericardial effusion Dullness to percussion at the point of maximal impulse Cardiomegaly without pulmonary congestion on chest X-ray Diagnosed with echocardiography-fluid within the pericardial sac ECG would show low voltage QRS complexes Tx: Acute pericardial effusion affecting hemodynamics - pericardiocentesis Chronic effusion and/or hemodynamically stable - treat underlying etiology, repeat pericardiocentesis or pericardiectomy may be indicated A blood pressure reading of 88/48 mm Hg (A) and jugular venous distension (D) are physical exam findings for a patient with pericardial tamponade. Although pericardial tamponade is fluid within the pericardial sac, it also includes a loss of diastolic function because the pressure from the fluid in the pericardial sac is much greater than the ventricular pressure. This pressure causes a collapse of the heart's ventricles. The typical presentation for those with pericardial tamponade includes hypotension, jugular venous distension, and muffled heart sounds. A pericardial knock (B) is consistent with the diagnosis of constrictive pericarditis, which occurs when the pericardium becomes thick, fibrotic, and calcified due to chronic episodes of inflammation. Secondary to these changes, there is restrictive ventricular filling, and a "knock" would be heard on auscultation when the blood hits the fibrotic cardiac walls.
A 60-year-old man presents with severe substernal chest pain, dyspnea, and diaphoresis that started one hour prior to the presentation. Pain radiates to his left neck and arm. His past medical history is significant for hypertension and hypercholesterolemia. On exam, he is noted to be tachycardic, diaphoretic, and hypertensive. Which of the following is the most appropriate next step in diagnosis? AChest radiography BEchocardiography CElectrocardiography DNuclear scintigraphy
CElectrocardiography STEMI Patient presents with substernal chest pain that radiates to the neck and arm****
A 58-year-old cachectic man with a 40 pack-year history of tobacco use presents to the clinic with severe shortness of breath and a cough. He is tachypneic and is using accessory muscles to breathe. A chest X-ray shows flattened diaphragm and an increase in anteroposterior diameter. Which of the following would you expect to find on this patient's pulmonary function test? ADecreased expiratory reserve volume BDecreased total lung capacity CFEV1/FVC less than 70% DIncreased diffusing capacity of the lungs for carbon monoxide
CFEV1/FVC less than 70% pulmonary function test, which would be consistent with obstructive disease. The patient's forced expiratory volume in one second (FEV1) would be decreased, however, the forced vital capacity (FVC) would stay the same, which causes a significant drop to the ratio of forced expiratory volume in one second to forced vital capacity (FEV1/FVC). restrictive lung dz= PFT=Decreased expiratory reserve volume (A) and decreased total lung capacity (B) Treatment options include bronchodilators, anticholinergics, steroids, supplemental oxygen, noninvasive ventilation, smoking cessation enlargement of the terminal airspaces accompanied by the destruction of alveolar walls, which leads to "air trapping." These patients are known as "pink puffers" because they breathe through pursed lips. They are commonly very thin and cachectic males with a positive history of smoking.
A 65-year-old man with a history of poorly controlled type 2 diabetes mellitus and hypertension presents to your office for his diabetes visit. Routine laboratory testing reveals elevated serum creatinine and blood urea nitrogen levels, increased from the previous year. Which of the following is the most appropriate diagnostic study to evaluate this patient? AIntravenous urogram BKidney computed tomography CKidney ultrasound DKidney, ureter, bladder radiography
CKidney ultrasound Kidney computed tomography (B) is often a complementary method of imaging to ultrasound but is typically not the initial imaging modality used. Use of contrast needs to be evaluated in the context of additional injury to the kidney, with risks and benefits being weighed Intravenous urogram (A) was used in the past as the main diagnostic tool in the evaluation of individuals with suspected kidney disease. This type of imaging can detect kidney stones as well as provide details about the shape, size, and anatomy of the kidneys. It requires contrast and also involves significant radiation so has been mostly replaced by computed tomography and ultrasound
A 35-year-old woman with a history of seizure disorder presents with an acute onset of seizure activity. She recently decreased the dosage of her seizure medications. Her seizures usually last less than 1 minute, but her family brought her in today because the seizure has lasted more than 5 minutes and still persists. On exam, bilateral tonic stiffening followed by rhythmic jerking of the limbs is noted. Which of the following is the most appropriate therapy indicated for this patient? AFosphenytoin BLevetiracetam CMidazolam DPhenobarbital
CMidazolam Status Epilepticus PE will show ≥ 5 minutes of continuous seizure activity When intravenous access is available, benzodiazepines (lorazepam and diazepam) are the first-line agents. In addition, nonbenzodiazepine antiseizure medication should be administered afterward to prevent the recurrence of seizures. Fosphenytoin or valproate is the preferred drug for most patients. When intravenous access is not available, IM** midazolam is a safe and effective alternative to the initial benzodiazepine administration. First-line: benzodiazepines (e.g., lorazepam) --> alternative IM midazolam Second-line: phenytoin or fosphenytoin, valproic acid, levetiracetam Third-line: pentobarbital, propofol, phenobarbital
A 66-year-old woman presents with a sensation of pulsation in her neck and abdomen. The patient reports that she has also had progressively worsening dyspnea on exertion and peripheral edema that began two months ago. She had a pacemaker placement nine months ago for a chronic bifascicular block. Physical exam is significant for distended, pulsatile neck veins, hepatomegaly, and 1+ generalized pitting edema. Palpation of the liver results in increased distension of the neck veins. Which of the following findings on physical exam would most likely correlate with the patient's condition? AHarsh midsystolic crescendo-decrescendo murmur radiating to the left shoulder BLoud midsystolic murmur best heard with the patient sitting and leaning forward CPansystolic murmur that becomes louder with inspiration DPansystolic murmur with prolonged apical impulse
CPansystolic murmur that becomes louder with inspiration Tricuspid regurgitation is a valvular disorder that occurs when there is retrograde blood flow from the right ventricle to the right atrium during systole. right-sided pressure overload leading to right-sided heart failure Pacemaker lead placement is an increasingly common iatrogenic cause of tricuspid regurgitation. As tricuspid regurgitation persists, right-sided cardiomegaly, systemic venous congestion, and eventually right-sided heart failure ensue. CARVALLO SIGN= pansystolic murmur that becomes louder with inspiration and reduced with expiration or Valsalva maneuver. It is best heard at the left lower sternal border and radiates to the right lower sternal border.
A 50-year-old woman is brought to the ED by her husband for sudden-onset nausea, abdominal pain, and blurred vision that began 30 minutes ago while she was sitting in the car at a drive-in movie theater. She reports that she saw halos around all the street lights on the way over to the ED. PE reveals a right eye with ciliary injection, a hazy cornea, and a mid-dilated, fixed pupil that is nonreactive to light. An intraocular pressure of 65 mm Hg is obtained (normal 12-22 mm Hg). A bolus of acetazolamide 500 mg IV brings the intraocular pressure down to 22 mm Hg. The patient is discharged with acetazolamide 250 mg PO four times daily and instructed to follow up with an ophthalmologist the following morning. Which of the following medications is most appropriate in the management of the patient's condition? ABrimonidine 0.2% ophthalmic solution BGlycerin PO 2 g/kg CPilocarpine 4% ophthalmic solution DPrednisolone acetate 1% eye drops
CPilocarpine 4% ophthalmic solution reverse angle closure=topical miotics=cause the pupil to contract (pilocarpine) reduce intraocular pressure=osmotic diuretics (mannitol, glycerol) Pilocarpine works by causing constriction of the pupillary sphincter muscle which pulls the iris away from the angle and allows the trapped aqueous fluid to flow out through the trabecular meshwork. Acute angle-closure glaucoma is a medical emergency that may result in permanent vision loss within two to five days of symptom onset if untreated. shallow anterior chamber or enlarged lens increases the risk of acute angle-closure glaucoma. report vision changes such as seeing halos around lights. Physical examination of the affected eye may reveal erythema, ciliary injection, a hazy or steamy cornea, and a mid-dilated fixed pupil that is nonreactive to light. Tonometry is useful in obtaining an intraocular pressure Initial treatment of acute angle-closure glaucoma involves rapidly decreasing the intraocular pressure. A bolus of acetazolamide 500 mg IV should be given immediately upon diagnosis. Osmotic diuretics (e.g., oral glycerin, IV urea, IV mannitol) may be used if patients are unresponsive to acetazolamide Definitive treatment of acute angle-closure glaucoma is surgical and includes laser peripheral iridotomy or surgical peripheral iridectomy acute unilateral pain and vision loss, headache, vomiting, and seeing halos around lights PE: cloudy cornea and fixed mid-dilated pupil
A 32-year-old man presents to the hospital with a recent history of new-onset dizziness. While interviewing the patient, he suddenly falls to the floor and starts to convulse with his eyes tightly shut!! His convulsions last for about two minutes. Which of the following most likely occurred? AClonic seizure BComplex partial seizure CPsychogenic seizure DTonic-clonic seizure
CPsychogenic seizure Often misdiagnosed as epileptic seizures, Psychological (e.g., emotional, stress-related) Neuro exam and workup will be normal Can fall under conversion disorder, factitious disorder, or malingering epileptic seizure- eye open during event, HA &incontinece common Complex partial seizures (B) typically start with an aura and proceed to an isolated neurologic manifestation.
A 38-year-old woman who is eight weeks postpartum presents to the office for a follow-up of an internal hemorrhoid that she has had for the past two months. She was seen about six weeks ago for the same problem and was told to increase her fiber and fluid intake. However, she continues to see bright red blood after passing stools and feels pressure to the anus that lasts for about one hour after every bowel movement. A physical exam reveals a pink, nontender lump extruding from the anus that is easily reducible with gentle pressure. She would like to explore other treatment options since she finds these symptoms very bothersome. Which of the following is the best next step in treatment for this patient? AHemorrhoidectomy BInfrared photocoagulation CRubber band ligation DSclerotherapy
CRubber band ligation Hemorrhoidectomy (A) is reserved for patients who have severe symptoms who have failed conservative nonsurgical treatments since it requires anesthesia and is difficult to recover from. Infrared photocoagulation (B) and sclerotherapy (D) can also be done in the office but are not the best answers since they are not as effective as rubber band ligation. Hemorrhoids Sx: discomfort and itching in the anal region, if thrombosed may also report pain PE: Internal: proximal to the dentate line External: distal to the dentate line Tx: lifestyle modifications, sitz baths, analgesic creams, rubber band ligation, sclerotherapy, surgical excision
A 35-year-old woman presents to a primary care provider with fatigue, arthralgias, and myalgias. Physical examination is significant for a malar rash. Laboratory testing reveals an antinuclear antibody titer of 1:640 and the presence of anti-Smith and anti-double-stranded DNA antibodies. The provider discusses the diagnosis and treatment plan with the patient and educates her on belimumab and hydroxychloroquine. Which of the following statements should be communicated to the patient? AAvoid live vaccines three months prior to beginning treatment BDiscontinue hydroxychloroquine if pregnant CSchedule an ophthalmologic examination within one year DShort-term high-dose IV corticosteroids should be avoided
CSchedule an ophthalmologic examination within one year Pharmacologic management includes antimalarials (e.g., hydroxychloroquine), nonsteroidal anti-inflammatory drugs, and corticosteroids. Because hydroxychloroquine may cause retinal toxicity, patients should schedule an ophthalmologic examination within one year of treatment initiation. Corticosteroids may be administered orally, topically, intramuscularly, or intravenously. Belimumab, approved in 2011, is the newest SLE medication and is a monoclonal antibody that inhibits the B lymphocyte stimulator Telling the patient that short-term high-dose IV corticosteroids should be avoided (D) is incorrect because IV corticosteroids are administered at high doses over a short period of time for patients with severe SLE complications such as nephritis or myocarditis. Long-term corticosteroid use is associated with adverse effects such as osteoporosis and cataracts
A 23-year-old woman presents to the emergency department four days after surgical repair of her anterior cruciate ligament. She has no significant past medical history, and her only medication is a combined oral contraceptive pill. She reports increased pain and swelling in her left leg. On exam, her left calf is erythematous, and its circumference is 4 cm greater than her right calf. What is the initial diagnostic study of choice for the suspected condition? AComputed tomography angiography BPlain radiography CVenous duplex ultrasound DVentilation-perfusion scan
CVenous duplex ultrasound DVT: venous stasis, endothelial damage, and hypercoagulability. Other risk factors include increased age, malignancy, pregnancy, oral contraceptives, and stroke Computed tomography venography is the definitive imaging modality if the ultrasound imaging is indeterminate. Anticoagulation therapy is the mainstay of treatment for DVT.
A 40-year-old Black man presents to a primary care provider for a follow-up visit to discuss his laboratory results. On the previous visit, a nontender, nodular prostate was palpated on digital rectal examination. His prostate-specific antigen level is 12.4 ng/mL. A transrectal ultrasound-guided biopsy is ordered. Which of the following is most likely present in the patient's history? ABone pain BPainless hematuria CWeak urinary stream DWeight loss
CWeak urinary stream prostate cancer! enlarged prostate due to prostate cancer or benign prostatic hyperplasia (BPH) causing compression of the urethra can result in a weak urinary stream nontender, nodular prostate on digital rectal examination is concerning for prostate cancer, while a smooth, boggy prostate with an obscured sulcus is associated with BPH. Bone pain (A) and weight loss (D) are both rare systemic symptoms associated with metastatic prostate cancer Painless hematuria (B) is the most common presenting symptom of bladder cancer but is not associated with early prostate cancer. Diagnosis is made by needle-core biopsy
A 55-year-old man presents with a gradual onset of unintentional weight loss, fatigue, and low-grade fever. On exam, splenomegaly is noted. Laboratory studies reveal a white blood cell count of 150,000/microL. The polymerase chain reaction of bone marrow aspirate demonstrates the presence of the BCR-ABL1 gene. Which of the following is the most likely diagnosis? CChronic lymphocytic leukemia DChronic myelocytic leukemia
DChronic myelocytic leukemia BCR-ABL1 gene=Philadelphia chromosome
Which of the following is the first serologic marker to be positive in acute hepatitis B? AHepatitis B core antibody BHepatitis B core antigen CHepatitis B surface antibody DHepatitis B surface antigen
DHepatitis B surface antigen Hepatitis B surface antibody (C) may not be detected for weeks to months after infection and, therefore, is not indicative of acute infection.
A 58-year-old man presents to a primary care provider with increasing right leg pain for the past four months. He reports that the pain is deep and achy and worse at night. The pain initially seemed to be localized to his upper thigh but is now also affecting his leg just above the knee. He also reports that his teeth have become more misaligned over the past few months and is wondering if that may be associated with the intermittent headaches he started having four months ago. He denies any falls, trauma, or changes in his daily activity. On physical examination, dilated scalp veins are present. Range of motion, strength, and stability of the lower extremities are within normal limits. A radiograph of the right femur is shown above. What is the most likely diagnosis? AOsteoarthritis BOsteochondritis dissecans COsteoid osteoma DPaget disease
DPaget disease lso known as osteitis deformans, is the second most common bone disease after osteoporosis. The underlying pathophysiology of Paget disease is an increase in osteoclast activity, resulting in lytic lesions followed by an increase in osteoblastic activity that causes disorganized bone formation. This results in highly vascularized, weak bones that are prone to deformation Labs: increased serum alkaline phosphatase and bone-specific alkaline phosphatase X-ray: bone thickening and enlargement with thickened cortices Tx: supportive, PT, bisphosphonates
A 57-year-old man presents to a primary care provider with fatigue and stocking-glove paresthesias. Physical examination reveals a smooth tongue, cheilosis, and abnormal vibration and joint position sense. A CBC shows leukopenia, anemia, and thrombocytopenia. The MCV is 120 fL. Hypersegmented neutrophils and macro-ovalocytes are present on the manual differential. Serum homocysteine and methylmalonic acid levels are increased. What is the most likely diagnosis? AAplastic anemia BFolate deficiency CG6PD deficiency DPernicious anemia
DPernicious anemia Pernicious anemia is the most common cause of vitamin B12 deficiency. +Neurologic changes include stocking-glove paresthesias, decreased vibratory and joint position sense, loss of fine touch sensation, loss of coordination, ataxia, or dementia. pernicious anemia and vitamin B12 deficiency may also have leukopenia and thrombocytopenia. Macrocytic anemia (MCV > 100 fL) is consistent with pernicious anemia, and the manual differential may show the presence of hypersegmented neutrophils and macro-ovalocytes. Both serum homocysteine and methylmalonic acid levels will be elevated. Folate deficiency (B) is a macrocytic anemia similar to pernicious anemia. The differentiating laboratory finding is methylmalonic acid, which is normal in folate deficiency. Unlike pernicious anemia, neurologic symptoms are absent.
A 22-year-old man with no past medical history presents to the clinic complaining of sudden-onset shortness of breath and right-sided pleuritic chest pain. He reports his symptoms "came out of nowhere." His vitals are as follows: weight 155 pounds, body mass index 21 kg/m2, O2 90% on room air, and blood pressure 120/86 mm Hg. His lung exam is consistent with decreased breath sounds, decreased tactile fremitus, and hyperresonance to percussion over the right upper lung field. The remainder of the lung exam is normal. Which of the following is the most likely diagnosis for this patient? AAsthma BPleural effusion CPneumonia DPneumothorax
DPneumothorax abnormal collection of air within the pleural space On lung exam, there will be decreased breath sounds, decreased tactile fremitus, and hyperresonance to percussion over the affected lung. A chest radiograph with expiratory view is the diagnostic test of choice and will show decreased peripheral lung markings. Pneumonia (C) will most commonly present with a cough and fever. On physical exam, there would be bronchial breath sounds, increased tactile fremitus, and dullness to percussion. Tx:< small ≤3 cm in a healthy patient: observation with oxygen administration> large >3 cm: needle aspiration or chest tube thoracostomy
A 63-year-old man presents to the clinic complaining of his "hands shaking." He notices whenever he is trying to do a specific task, such as reaching out to grab a drink or writing in his journal, his hands shake. The patient notes the tremor improves after drinking whiskey. On physical exam, there is no resting tremor noted, however, when performing the finger-to-nose exam, the patient's tremor increases as he approaches the target. The remainder of the neurologic exam is normal. Which of the following is the most appropriate first-line treatment for this patient's condition? AAlprazolam BCarbidopa CGabapentin DPropranolol
DPropranolol Essential tremor finger-to-nose test, the patient's tremor increases as the target approaches. Otherwise, the patient's neurological exam will have no other significant findings. 1) first-line medical treatment for an essential tremor is propranolol or primidone. 2) If either propranolol or primidone alone fails, a trial of both medications combined may be effective. 3) third-line treatment involves using either gabapentin, topiramate, or alprazolam.
Which of the following conditions is the most common cause of secondary hypertension in adults? AGraves disease BPheochromocytoma CPrimary aldosteronism DRenal artery stenosis
DRenal artery stenosis
A 68-year-old man presents to a primary care provider with complaints of fatigue, anorexia, insomnia, and muscle cramps. A complete laboratory workup is ordered and is significant for a hemoglobin of 9.1 g/dL, potassium of 5.4 mmol/L, phosphate of 1.8 mmol/L, and glomerular filtration rate of 29 mL/min/1.73 m2. Which of the following dietary modifications is most appropriate for this patient? AIncreased phosphate intake to > 1,000 mg/day BIncreased water intake to > 2 L/day CRestricted calcium intake to < 1,000 mg/day DRestricted protein intake to 0.8 g/kg/day
DRestricted protein intake to 0.8 g/kg/day stage 4 chronic kidney disease (CKD) and should be evaluated by a renal nutritionist. A restricted protein intake to 0.8 g/kg/day is recommended as part of his dietary management to slow CKD progression. Patients with chronic kidney disease should have an adequate caloric intake, calcium and vitamin D supplementation, and limitation of fluids, sodium, potassium, and phosphorus.
A 45-year-old woman presents to the clinic. She appears quite somnolent and states she is in pain. She reports pain in her hands, forearms, and knees, along with abdominal discomfort. She also describes sensitivity to cold in her hands, which she says makes her hands turn white and become painful. She reports her symptoms have been present on and off for the last few years but have begun to significantly worsen. Her medical history is significant for hypertension and interstitial lung disease with home oxygen use. The patient reports no tobacco or illicit drug use. She takes lisinopril, metoprolol, hydrochlorothiazide, and mycophenolate. Physical exam reveals puffy, thickened skin over the fingers on both hands, tenderness over multiple joints, rales, labored respirations, general body atrophy, abdominal rigidity, and a systolic regurgitation murmur of grade 3 over the tricuspid area. Her vitals are a temperature of 99.5°F, HR of 95 bpm, RR of 20/minute, BP of 140/100 mm Hg, oxygen saturation of 93%, and body mass index of 14 kg/m2. Which of the following is the most likely diagnosis? AAcromegaly BPolymyalgia rheumatica CPolymyositis DScleroderma
DScleroderma localized vs systemic (2 types) THICKENING/HARDENING OF SKIN** Systemic sclerosis (scleroderma) is classically defined as a chronic, progressive, fibrotic disease that affects multiple systems. Broadly, it commonly involves vascular dysfunction (leading to ischemia) and fibrosis of skin and internal organs. CREST (calcinosis, Raynaud phenomenon, esophageal dysmotility, sclerodactyly, and telangiectasia). skin thickening with rapid progression, including lung, kidney, and cardiac involvement. Anti-topoisomerase I (anti-Scl-70) antibody (specific for diffuse disease) anticentromere antibody (specific for limited disease)
A nonsymptomatic, overweight, 68-year-old smoker with a history of diabetes, hypertension, and hyperlipidemia undergoes a screening ultrasound for an abdominal aortic aneurysm. Screening reveals an abdominal aortic aneurysm with a diameter of 3 cm. Which recommendation would result in the greatest immediate reduction of risk for this patient? AAdding an angiotensin-converting enzyme inhibitor BInitiating a diet and exercise routine CInitiating statin therapy DSmoking cessation
DSmoking cessation Abdominal Aortic Aneurysm (AAA) Risk factors: male sex, older patients, smoking, HTN Abdominal pain or asymptomatic Physical exam will show pulsatile abdominal massIf ruptured: hypotension Diagnosis is made by US: excellent screening tool ManagementMonitor progression (Society for Vascular Surgery guidelines) 4.0-4.9 cm: US annually 5.0-5.4 cm: US every 6 months, can also use CT or MRI (MRI is preferred over time due to less radiation) Surgical repair = symptomatic> 5.5 cm or aneurysms with rapid expansion rate: elective surgery
A 61-year-old woman presents to the office with worsening flu-like symptoms for five weeks. She reports low-grade fever most days, general malaise, and night sweats. On physical exam, you notice painless erythematous macules on the palms and soles, as well as tender nodules on the pads of the fingers. Laboratory results reveal leukocytosis and anemia, blood cultures show gram-positive cocci, and the blood agar shows a green discoloration. What is the most common pathogen responsible for this patient's symptoms? AEnterococcus BStaphylococcus aureus CStreptococcus pyogenes DStreptococcus viridans
DStreptococcus viridans Bacterial endocarditis=nfective endocarditis Streptococcus viridans is the most common pathogen responsible for subacute bacterial endocarditis. Additionally, Streptococcus viridans is the most common mouth pathogen and may enter the bloodstream following a dental procedure or tooth decay. Acute bacterial endocarditis is an infection that is rapidly destructive and typically fatal if not successfully treated within six weeks. Subacute bacterial endocarditis is an indolent infection and can be fatal if not successfully treated within one year Staphylococcus aureus -IVDU and Streptococcus pyogenes are the most common pathogens responsible for acute bacterial endocarditis.
A 55-year-old man with a history of hypertension presents to the emergency department for a headache that has lasted three days. The patient reports that the headache started suddenly, is unilateral, and is associated with photophobia. The patient also reports a stiff neck. Vital signs are within normal limits. A noncontrast CT scan is performed and the radiologist reports "no acute intracranial abnormalities." A lumbar puncture reveals an opening pressure of 28 cm H2O and a clear dark yellow appearance of the CSF. Which of the following is the most likely diagnosis? AInfectious meningitis BMigraine CPseudotumor cerebri DSubarachnoid hemorrhage
DSubarachnoid hemorrhage sudden-onset "worst headache of (their) life." Infectious meningitis (A) is the result of viral or bacterial invasion and colonization of the cerebrospinal fluid (CSF). Patients will typically present with a headache, signs of meningeal inflammation, and a fever. CSF fluid analysis will show an increased white blood cell count, an elevated opening pressure, a low glucose concentration, and an elevated protein level. blood will appear WHITE on noncontact CT scan. Noncontrast CT scan is diagnostic in the majority of cases but can sometimes be negative. If CT scan is negative but the clinical suspicion is high for subarachnoid hemorrhage, a lumbar puncture should be performed. The presence of xanthochromia (which is a yellow!!! or pink appearance of the cerebrospinal fluid representing broken down red blood cells) or gross blood in the cerebrospinal fluid is diagnostic for subarachnoid hemorrhage After diagnosis is made, cerebral angiogram should be done to locate the site of the aneurysm. Initial treatment is achieved with surgical clipping or coiling the aneurysm. Treatment is supportive and nimodipine=CCB (decreases vasospasm)
A 45-year-old Caucasian man presents to the office for a routine checkup and is found to have elevated serum iron levels. He complains of some fatigue but has no other symptoms. A physical exam is negative. A transferrin saturation level is ordered, which is elevated at 17 ng/mL, and a genetic test shows the presence of a mutation in the HFE gene. Which of the following is the best initial treatment of choice for this patient? ADietary changes to avoid organ meats and alcoholic beverages BIron chelation CTherapeutic erythrocytapheresis DTherapeutic phlebotomy
DTherapeutic phlebotomy Hereditary hemochromatosis is a genetic disorder caused by a mutation of the hemochromatosis HFE protein, resulting in an iron overload state. It is considered a disease of the liver, is a common autosomal recessive genetic disorder will have hepatomegaly, skin hyperpigmentation, and signs of arthritis (e.g., joint tenderness and swelling). elevated serum iron levels are found incidentally on a routine chemistry screening panel. The initial test of choice is transferrin saturation level, which will be elevated. Genetic testing for the HFE mutation, serum ferritin studies, and hepatic iron concentration are also helpful plasma ferritin level falls in the low normal range (5-10 ng/mL). Once stable, patients should receive phlebotomy every two to four months for maintenance. Iron chelation, with agents such as deferoxamine, is used when phlebotomy is contraindicated
A 35-year-old woman with central obesity, a large hump behind her neck, striae of her skin, and easy bruising presents to the health clinic for worsening proximal muscle weakness. She is noted to be hypertensive, has gained weight, and has darkening of the skin on the back of her neck with a velvety texture. She is also noted to have hirsutism and acne on her face. Laboratory studies show a normal glucose level and hypokalemia. High-dose dexamethasone suppression test shows ACTH suppression, and an MRI shows a pituitary adenoma. What is the best clinical intervention for this patient, based on the most likely diagnosis? AAdrenalectomy BKetoconazole CSteroid taper DTranssphenoidal surgery
DTranssphenoidal surgery=remove pituitary adenoma Cushing disease is caused by excess ACTH produced most commonly by a benign pituitary adenoma. Increased ACTH causes excess cortisol and glucocorticoid production Symptoms include truncal obesity, moon facies, buffalo hump, wasting of extremity muscles, proximal muscle weakness, and striae of the abdome Diagnosis of Cushing disease is made by high-dose dexamethasone suppression test, which will show ACTH suppression, ruling out other causes of increased ACTH or cortisol production. MRI of the brain can show a pituitary mass, which is suggestive of Cushing disease. Adrenalectomy (either surgical or medical with mitotane) is a treatment choice for Cushing disease that is refractory to transsphenoidal surgery MCC of Cushing syndrome? Long-term high-dose corticosteroid therapy.
A 35-year-old man presents with epigastric pain, which radiates to the back, accompanied by nausea and vomiting. On exam, he is noted to have cream-colored retinal vessels. Laboratory studies demonstrate elevated amylase and lipase. Which of the following findings is most likely to also be elevated? and diagnosis? AHigh-density lipoprotein BLipoprotein(a) CLow-density lipoprotein DTriglyceride
DTriglyceride Hyperlipidemia Severe elevation in triglycerides can cause pancreatitis total cholesterol (normal: < 200 triglycerides (normal: < 150 HDL cholesterol (protective: > 60 LDL cholesterol (optimal: < 100 mg/dL
A 65-year-old man presents with acute onset of severe muscle cramps and extremity spasms. QT prolongation is seen on ECG. Laboratory studies are remarkable for a serum calcium level of 7.3 mg/dL. Carpopedal spasm is witnessed with inflation of the sphygmomanometer. Which of the following best describes this physical exam finding associated with the underlying diagnosis? ABand keratopathy BChvostek sign CDecreased deep tendon reflex DTrousseau sign
DTrousseau sign=more specific Hypocalcemia occurs when the serum calcium level falls below 8.5 trousseau sign (carpal spasm after blood pressure cuff is applied for three minutes) and Chvostek sign (spasm of facial muscle after tapping the facial nerve in front of the ear). chvostek sign (B) is a classic sign seen in approximately 10% of patients with normal serum calcium and, hence, is less specific than Trousseau sign. It is facial spasms that result when the facial nerve is percussed.
A 20-year-old unvaccinated college student is brought in by his roommate to the college infirmary for altered mental status. The roommate reports the patient began complaining of fever, headache, and stiff neck yesterday. This morning, he was found in bed and was difficult to wake up. On evaluation, the patient only moans to painful stimuli, has a fever of 101.7°F, and is unable to put his chin to his chest. He has a diffuse purple rash that is nonblanching. You perform a lumbar puncture. What are the most likely cerebrospinal fluid results? ABloody color, elevated opening pressure, normal lymphocytes, normal protein, and normal glucose BClear color, high opening pressure, normal lymphocytes, normal protein, and normal glucose CClear color, normal opening pressure, normal lymphocytes, normal protein, and normal glucose DTurbid color, elevated opening pressure, elevated lymphocytes, elevated protein, and low glucose
DTurbid color, elevated opening pressure, elevated lymphocytes, elevated protein, and low glucose Bacterial meningitis is a central nervous system infection most commonly caused by Streptococcus pneumoniae, Neisseria meningitidis, or group B streptococci. SYM:altered mental status, fever, headache, vomiting, and stiff neck, however, not all symptoms may be present.
A 35-year-old patient presents to the hospital following a motor vehicle collision. When you arrive in the exam room, the patient is unresponsive. Which of the following exam findings most likely indicates early uncal herniation? AAbsent oculovestibular reflex BBilateral pinpoint pupilsYour Answer CGaze preference away from the lesion DUnilateral dilated and fixed pupil
DUnilateral dilated and fixed pupil The two most common herniation syndromes are central herniation (caused by a mass that directs a vertical vector of force toward the tentorial notch) and uncal herniation. Uncal herniation results from a unilateral mass causing a lateral force vector. This force vector pushes the temporal lobe over the edge of the tentorium and compresses the midbrain. Coma can be defined as a depressed level of consciousness that impairs one's ability to react to or sense external stimuli. In the case of uncal herniation, only the ipsilateral eye will be affected, resulting in unilateral dilated and fixed pupil, Bilateral pinpoint pupils (B) is a common finding in patients with opioid overdose. It is not a finding associated with uncal herniation.
A 50-year-old man presents to his primary care provider with complaints of fatigue, night sweats, weight loss, and bone pain. Laboratory testing is significant for a normochromic, normocytic anemia. The peripheral blood smear is shown in the image above. A comprehensive metabolic panel is significant for a blood urea nitrogen of 50 mg/dL, a creatinine of 3.5 mg/dL, and a total protein of 14.7 g/dL. Bence Jones proteins are present in the urine. Which of the following pathologic fractures is most commonly associated with the patient's diagnosis? AAcetabular fracture BHumeral neck fractureYour Answer CHumeral shaft fracture DVertebral fracture
DVertebral fracture most common pathological fractures associated with multiple myeloma are vertebral, pelvic, femoral neck, and rib fractures. Multiple myeloma, also known as plasma cell myeloma, is a primary malignant bone tumor that occurs due to the hyperproliferation of plasma cells, which produce monoclonal immunoglobulins Back pain Signs and symptoms: CRAB: hypercalcemia, renal insufficiency, anemia, lytic bone lesions or back pain Serum protein electrophoresis: M spike Protein electrophoresis urine analysis: Bence Jones proteins
A 74-year-old man with a past medical history of hypertension presents to the emergency department for sudden-onset confusion and a coarse tremor. The patient presents with altered mental status, and on physical exam, you notice significant myoclonus of the upper extremities as well as muscle rigidity, postural instability, and bradykinesia. Vitals are heart rate 92 bpm, blood pressure 142/79 mm Hg, respiratory rate of 16 breaths per minute, O2 saturation of 97% on room air, and temperature of 98.7°F. His daughter reports the pair recently returned from a camping trip, where the patient complained of being bitten by multiple mosquitos. What is the most likely diagnosis? ABacterial meningitis BLyme disease CRocky Mountain spotted fever DViral encephalitis
DViral encephalitis West Nile encephalitis is characterized by the viral infiltration of the central nervous system, resulting in acute neurologic symptoms. Almost all cases of West Nile virus infection are caused by the virus being transmitted through a mosquito bite. Most patients infected with West Nile virus are asymptomatic, however, symptoms of West Nile encephalitis include altered mental status ranging from subtle deficits to complete unresponsiveness. Focal neurologic deficits such as flaccid paralysis, coarse tremor, rigidity, postural instability, and bradykinesia can also be observed. Diagnosis is made via cerebrospinal fluid (CSF) examination using enzyme-linked immunosorbent assay (ELISA). tx-supportive, with many patients regaining their baseline motor function in six to eight weeks after symptom onset. Some therapies that have been suggested for the treatment of West NIle encephalitis are interferon, ribavirin, and IVIG, however, none of these therapies have been shown to improve patient outcomes and, in some cases, have been harmful to the patient. Bacterial meningitis (A) would present with fever, headache, neck stiffness, and a normal neurologic exam. Lyme disease (B) would present initially with myalgias and fever and would progress to a meningitis-like clinical picture without acute seizures or altered mental status.
Chronic Bronchitis Patient will be overweight and cyanotic (blue bloater) Chronic productive cough for at least 3 months in at least 2 successive years PE will show decreased breath sounds, increased resonance upon percussion of the lung fields Diagnosis is made by FEV1/FVC ratio < 0.7 and FEV1 < 80% predicted
Emphysema (C) is the other form of chronic obstructive lung disease, however, it is typically seen in patients who are thin. Patients with emphysema typically do not present with a cough, rather their main complaint is shortness of breath. Idiopathic pulmonary fibrosis (D) presents with a restrictive physiology on pulmonary function test and rarely, may improve after administration of albuterol, dyspnea is a prominent feature rather than cough with sputum production.
What laboratory testing should be reviewed before beginning metformin?
Hemoglobin A1C, liver function tests, and serum creatinine.
bac meningitis tx
Immunocompetent children and adults aged 1 month to 50 years should be treated with cefotaxime or ceftriaxone plus vancomycin. Adults older than 50 years, patients with a history of alcohol use disorder, or patients with debilitating illness should also receive ampicillin. The addition of dexamethasone is recommended for Streptococcus pneumoniae infections.
iron deficiency lab?
Iron Deficiency Anemia Sx: weakness, dizziness, and fatigue PE: pallor, tachycardia, atrophic glossitis, or koilonychia (spoon nails) Labs: microcytic, hypochromic red blood cells, decreased serum iron level, an increase in the total iron binding capacity (TIBC), and decreased serum ferritin levels Most common cause of microcytic anemia
two classic meningeal signs?
Kernig and Brudzinski signs.
acute kidney injury labs? inc or dec in serum creatinine, blood urea nitrogen (BUN) to creatinine ratio, UA, fractional excretion of Na+ (FENa+), urine osmolality and Na+
Labs will show increase in serum creatinine, blood urea nitrogen (BUN) to creatinine ratio, UA, fractional excretion of Na+ (FENa+), urine osmolality and Na+
Gout lab findings?
Mono or oligo-articular arthritis caused by uric acid crystals Risk factors: male sex, age > 30 Sx: podagra (acute onset of pain in the first MTP) PE: hot, red, tender joint, tophi Labs: needle-shaped crystal with negative birefringence, uric acid can be low, normal or elevated TreatmentAcute: NSAIDs, steroids, colchicineChronic: allopurinol (first line), febuxostat, probenecid Can be triggered by loop and thiazide diuretics
A 46-year-old man presents to his primary care provider with a headache, shortness of breath, and palpitations. He reports that his symptoms started after a long weekend of binge drinking. He typically drinks seven beers per day. He has a 35 pack-year history and quit smoking five years ago when he was diagnosed with COPD. He has a history of hypertension and is currently taking lisinopril 40 mg PO daily. His family history is significant for a father who passed away from a myocardial infarction at 44 years of age and a brother who had a stroke at 32 years of age. His rhythm strip is shown above. Which of the following is the most likely underlying cause of the patient's dysrhythmia? EKG=MAT AChronic obstructive pulmonary disease BExcessive alcohol use CFamily history of coronary artery disease DHypertension
Multifocal Atrial Tachycardia Associated with older patients and those with COPD Rate will be 100-200 beats/min PR interval will differ Notable feature: at least three different P wave forms Treatment is to treat the underlying cause, calcium channel blockers Excessive alcohol use (B) and alcohol withdrawal are most closely associated with "holiday heart" or AFIB. It is also the most likely cause of the patient's headache in this case. The patient's family history of coronary artery disease (C) puts him at increased risk for myocardial infarction, but COPD is more likely to cause MAT than coronary artery disease. Hypertension (D) may increase atrial automaticity and increases the risk of dysrhythmias, including atrial fibrillation and multifocal atrial tachycardia, but COPD is a much more common cause of MAT.
Pleural Effusion PE? CXR
PE will show ↓ breath sounds + dull percussion + ↓ tactile fremitus CXR will show blunting of the costophrenic angle Can also use CT or US to diagnose Most common causes: Transudate: heart failure Exudate: infection > malignancy, PE Management includes treating underlying cause, therapeutic thoracentesis, tube thoracostomy Light criteria are used to differentiate between transudative and exudative effusions
Cirrhosis Most commonly caused by alcohol, hepatitis, autoimmune
PE: gynecomastia, palmar erythema, ↑ bleeding Hepatic encephalopathy: asterixis, confusion Portal HTN: caput medusae, splenomegaly, ascites ↑ risk for HCC, screen with ultrasound +/- AFP every 6 months
polymyalgia rheumatica vs. polymyositis
Polymyalgia rheumatica (B) is a rheumatic condition involving symmetric ache and stiffness of more proximal musculoskeletal regions, such as shoulders, hips, neck, and torso, especially in the morning. It does not have skin, lung, or cardiac manifestations. Polymyositis (C) can present with interstitial disease, cardiac involvement, myalgias, and other similar presentations to systemic sclerosis but usually does not involve skin thickening. It is characterized by muscle weakness but not pain.
? will result in middle or lower lobe consolidation on chest X-ray. and TX is?
Primary TB Active TB is treated with the RIPE regimen. This includes rifampin, isoniazid, pyrazinamide, and ethambutol.
differentiate types of bacterial meningitis
Streptococcus pneumoniae is one of the most common causative agents of bacterial meningitis in neonates younger than 1 month. Streptococcus pneumoniae and Neisseria meningitidis are the most common causative agents in adults, Listeria monocytogenes can be present in patients over 50 years of age or those who have deficiencies in cell-mediated immunity. Neisseria meningitidis infections can have a petechial rash and palpable purpura as well.
tension HA tx? abortive vs. preventative
Treatment is NSAIDs (abortive), TCAs (preventive) prophylactically treated using antidepressants such as amitriptyline or selective serotonin reuptake inhibitors.
Displaced point of maximal impulse (A) is a sign of ? Muffled heart sounds (B) are common in ? also present with dyspnea and peripheral edema. An echocardiogram would reveal the presence of an effusion and evidence of diastolic collapse of heart chambers. S3 gallop (D) is commonly seen in patients with? This extra sound is due to blood striking a compliant ventricle.
cardiomegaly pericardial tamponade heart failure with reduced ejection fraction
pulmonary function test for a patient with restrictive lung disease show?
normal or increased FEV1/FVC, a decreased total lung capacity, and a decreased pulmonary reserve volume. ↓FVC, ↓TLC and normal or increased FEV1/FVC ratio, ↓DLCO
A 55-year-old woman presents to the clinic with acute onset of cough, shortness of breath, and fever. CXR-infiltrates ADecreased tactile fremitus BDiffuse end-expiratory wheezing CDullness to percussion of the right lower lobe DHyperresonance to percussion of the right lower lobe
pneumonia CDullness to percussion of the right lower lobe CXR - interstitial infiltrates and lobar opacity On pulmonary exam, dullness to percussion, decreased breath sounds, increased tactile fremitus, and rales Patients who are admitted to the hospital and treated on a general medical floor are often treated with a beta-lactam agent, such as ceftriaxone** or cefotaxime, and a macrolide, such as azithromycin**. CAP treated outpatient may include one of the following: combination therapy with a beta-lactam antibiotic (amoxicillin, amoxicillin-clavulanic acid, cefuroxime, or cefdinir) and a macrolide antibiotic (azithromycin), doxycycline monotherapy, or a fluoroquinolone monotherapy (levofloxacin).
A harsh midsystolic crescendo-decrescendo murmur radiating to the left shoulder (A) and neck that is best heard at the second to third left intercostal space is associated with ? A loud midsystolic murmur best heard with the patient sitting and leaning forward (B) is associated with ?. best heard at the second right intercostal space and radiates to the neck and left sternal border. A pansystolic murmur with prolonged apical impulse (D) that is best heard at the apex and radiating to the left axilla is associated with ?
pulmonic stenosis-harsh midsystolic crescendo-decrescendo murmur radiating to the left shoulder aortic stenosis-loud midsystolic murmur best heard with the patient sitting and leaning forward &right intercostal space and radiates to the neck and left sternal border. mitral regurgitation- pansystolic murmur with prolonged apical impulse& at apex and radiating to the left axilla
what med cause erectile dysfunction?
serotonin and norepinephrine reuptake inhibitors (e.g., duloxetine, venlafaxine), tricyclics (e.g., amitriptyline, nortriptyline), and older monoamine oxidase inhibitors (e.g., isocarboxazid, phenelzine)
Statins act by inhibiting hydroxymethylglutaryl-coenzyme A (HMG-CoA). Once statin therapy is initiated, what test? should be monitored every six weeks until goals are met
serum lipid levels
Bloody color, elevated opening pressure, normal lymphocytes, normal protein, and normal glucose (A) ? Clear color, high opening pressure, normal lymphocytes, normal protein, and normal glucose (B) ? Clear color, normal opening pressure, normal lymphocytes, normal protein, and normal glucose (C) ?
subarachnoid bleed. idiopathic intracranial hypertension viral or aseptic meningitis.
Pharmacotherapeutics that can cause or exacerbate orthostatic hypotension (dizziness when standing up for 15 min) include? not often experience dizziness while sitting or walking, but upon standing, she feels very weak and often needs to sit for 10 minutes before she attempts to stand again. systolic dec 20, diastolic dec 10
terazosin, antidepressant drugs, antihypertensive drugs, beta-blockers (specifically propranolol), diuretics (specifically hydrochlorothiazide and furosemide), vasodilators (specifically hydralazine and calcium channel blockers), narcotics, and alcohol. drug has been eliminated or changed, treatment should then focus on nonpharmacologic measures such as increasing salt intake, initiating use of compression stockings, introducing physical maneuvers, and exercising. If nonpharmacologic measures are ineffective at appropriately managing orthostatic hypotension, pharmacotherapeutics, such as low-dose fludrocortisone for patients with volume depletion, or a sympathomimetic pressor, such as midodrine, may be considered
USPSTF recommends one-time screening for AAA by? what and what age? F/M?
ultrasonography in men aged 65-75 who have ever smoked